Normalizing factor of wave function

  • #1
Lagrange fanboy
9
2
TL;DR Summary
Confusion over an equation on page 256 of Quantum Mechanics - The Theoretical Minimum
So on page 256 of Quantum Mechanics - The Theoretical Minimum, it says that the wave function of a momentum eigenvector, with respect to the position eigenbasis is ##\psi_p(x)=Ae^{\frac{ipx}{\hbar}}##, and ##A## must be ##\frac{1}{\sqrt{2\pi}}## to keep it a unit vector. However why must ##A=\frac{1}{\sqrt {2\pi}}##? ##\bar \psi_p=\bar Ae^{-\frac{ipx}{\hbar}}##, so ##| \psi_p|^2=A\bar A=|A|^2##. Plugging in ##A=\frac{1}{\sqrt{2\pi}}## that means ##\int_{-\infty}^\infty \psi_p\bar\psi_p dx= \int_{-\infty}^\infty \frac{1}{2\pi}dx\not = 1##
 
Physics news on Phys.org
  • #2
Lagrange fanboy said:
TL;DR Summary: Confusion over an equation on page 256 of Quantum Mechanics - The Theoretical Minimum

So on page 256 of Quantum Mechanics - The Theoretical Minimum, it says that the wave function of a momentum eigenvector, with respect to the position eigenbasis is ##\psi_p(x)=Ae^{\frac{ipx}{\hbar}}##, and ##A## must be ##\frac{1}{\sqrt{2\pi}}## to keep it a unit vector. However why must ##A=\frac{1}{\sqrt {2\pi}}##? ##\bar \psi_p=\bar Ae^{-\frac{ipx}{\hbar}}##, so ##| \psi_p|^2=A\bar A=|A|^2##. Plugging in ##A=\frac{1}{\sqrt{2\pi}}## that means ##\int_{-\infty}^\infty \psi_p\bar\psi_p dx= \int_{-\infty}^\infty \frac{1}{2\pi}dx\not = 1##
The integral you have is divergent, using normal calculus. In QM, however, you can appeal to the following identity:$$\delta(p) = \frac 1 {2\pi}\int_{-\infty}^{+\infty} e^{ipx} \ dx$$
 
  • Like
Likes vanhees71
  • #3
Lagrange fanboy said:
TL;DR Summary: Confusion over an equation on page 256 of Quantum Mechanics - The Theoretical Minimum

So on page 256 of Quantum Mechanics - The Theoretical Minimum, it says that the wave function of a momentum eigenvector, with respect to the position eigenbasis is ##\psi_p(x)=Ae^{\frac{ipx}{\hbar}}##, and ##A## must be ##\frac{1}{\sqrt{2\pi}}## to keep it a unit vector. However why must ##A=\frac{1}{\sqrt {2\pi}}##? ##\bar \psi_p=\bar Ae^{-\frac{ipx}{\hbar}}##, so ##| \psi_p|^2=A\bar A=|A|^2##. Plugging in ##A=\frac{1}{\sqrt{2\pi}}## that means ##\int_{-\infty}^\infty \psi_p\bar\psi_p dx= \int_{-\infty}^\infty \frac{1}{2\pi}dx\not = 1##
Actually, it turns out that the "wavefunction" ##e^{ipx/\hbar}## can't be renormalized, as you can see from your calculation. The plane waves don't belong to Hilbert space, so we can "normalize" them using any convention we like. The one you mentioned follows the definition of a Fourier transform.

-Dan
 
  • Like
Likes vanhees71
  • #4
PeroK said:
The integral you have is divergent, using normal calculus. In QM, however, you can appeal to the following identity:$$\delta(p) = \frac 1 {2\pi}\int_{-\infty}^{+\infty} e^{ipx} \ dx$$
But the norm of the wavefunction won't be 1 then, shouldn't this cause a problem down the line when calculating probabilities?
 
  • Like
Likes vanhees71
  • #5
Lagrange fanboy said:
the norm of the wavefunction won't be 1 then, shouldn't this cause a problem down the line when calculating probabilities?
The wave function ##e^{ipx}## is not physically realizable, so the fact that it is not normalizable doesn't cause any actual issue. If you want a heuristic probability interpretation of it, it says that the particle is equally likely to be at any ##x## value (i.e., position); some sources will describe this as the position being "completely uncertain". (Conversely, a position eigenstate, ##\delta(x)##, can be described as the momentum being "completely uncertain".) But in practice we never need to compute probabilities for such states, only for linear combinations of them (wave packets) that are normalizable.
 
  • Like
Likes topsquark
  • #6
Lagrange fanboy said:
But the norm of the wavefunction won't be 1 then, shouldn't this cause a problem down the line when calculating probabilities?
These wavefunctions are not physical in themselves, but occur in continuous wave packets. The point of the normalisation constant is to provide an ersatz orthonormality:$$\braket{\psi_p |\psi_q} = \delta(p - q)$$I don't have Susskind's book, so I don't know how and where he explains all this.
 
  • Like
Likes topsquark
  • #7
PeterDonis said:
The wave function ##e^{ipx}## is not physically realizable, so the fact that it is not normalizable doesn't cause any actual issue. If you want a heuristic probability interpretation of it, it says that the particle is equally likely to be at any ##x## value (i.e., position); some sources will describe this as the position being "completely uncertain". (Conversely, a position eigenstate, ##\delta(x)##, can be described as the momentum being "completely uncertain".) But in practice we never need to compute probabilities for such states, only for linear combinations of them (wave packets) that are normalizable.
Still, if ##Ae^{\frac{ipx}{\hbar}}## is the wave function for ##|p\rangle## wrt. position basis, then I'd expect ##\int_{-\infty}^\infty \langle p|x\rangle \langle x|p\rangle dx = 1##.
 
  • #8
Lagrange fanboy said:
Still, if ##Ae^{\frac{ipx}{\hbar}}## is the wave function for ##|p\rangle## wrt. position basis, then I'd expect ##\int_{-\infty}^\infty \langle p|x\rangle \langle x|p\rangle dx = 1##.
Then your expectation in this case is simply wrong. This expectation is only valid for wave functions that are physically realizable. As I've already said, ##e^{ipx}## is not physically realizable.
 
  • Like
Likes topsquark
  • #9
Lagrange fanboy said:
Still, if ##Ae^{\frac{ipx}{\hbar}}## is the wave function for ##|p\rangle## wrt. position basis, then I'd expect ##\int_{-\infty}^\infty \langle p|x\rangle \langle x|p\rangle dx = 1##.
In QM you often have to expect the unexpected!
 
  • #10
Lagrange fanboy said:
Still, if ##Ae^{\frac{ipx}{\hbar}}## is the wave function for ##|p\rangle## wrt. position basis, then I'd expect ##\int_{-\infty}^\infty \langle p|x\rangle \langle x|p\rangle dx = 1##.
Quantum Mechanics pretty much only uses wavefunctions (or bras and kets if you are doing Matrix Mechanics), but all of these live in something called "Hilbert space." The function ##e^{i p x/\hbar}## does not actually have a defined norm, so it does not belong to Hilbert space, and thus we don't actually call it a wavefunction. (It often is called a wavefunction, though, even by the instructors.) It is often included along with wavefunctions, though, because it is a useful way to express certain types of behavior.

You just have to keep in mind that the plane wave solutions aren't actually wavefunctions and behave slightly differently.

-Dan
 
  • #11
topsquark said:
Quantum Mechanics pretty much only uses wavefunctions (or bras and kets if you are doing Matrix Mechanics), but all of these live in something called "Hilbert space." The function ##e^{i p x/\hbar}## does not actually have a defined norm, so it does not belong to Hilbert space, and thus we don't actually call it a wavefunction. (It often is called a wavefunction, though, even by the instructors.) It is often included along with wavefunctions, though, because it is a useful way to express certain types of behavior.

You just have to keep in mind that the plane wave solutions aren't actually wavefunctions and behave slightly differently.

-Dan
Wow now I'm really confused. I read on wikipedia that the postulates of quantum mechanics include:
  • Quantum states are unit vectors in some Hilbert space
  • Observables are Hermitian operators acting on said Hilbert space
  • Probability of results are calculated from inner products between the system's current state and the observable's eigenstates
Since ##e^{\frac{ipx}{\hbar}}## isn't in the Hilbert space does that mean the postulates above are inconsistent? What are the correct postulates then?
 
  • #12
Lagrange fanboy said:
Wow now I'm really confused. I read on wikipedia that the postulates of quantum mechanics include:
  • Quantum states are unit vectors in some Hilbert space
  • Observables are Hermitian operators acting on said Hilbert space
  • Probability of results are calculated from inner products between the system's current state and the observable's eigenstates
Since ##e^{\frac{ipx}{\hbar}}## isn't in the Hilbert space ...
... it means it isn't a quantum state. In other words, momentum has no eigenstates. It has eigenfunctions outside the Hilbert space, which are mathematically useful, but not themselves quantum states. I'm surprised if Susskind has not said all this.
 
  • Like
Likes vanhees71 and topsquark
  • #13
Lagrange fanboy said:
I read on wikipedia
Which is not a good primary source for understanding science, particularly not when a fairly subtle technical issue like this one comes up.

If we were to sharpen up those postulates to take into account the technical issue under discussion, there are at least two ways to do it. One way, the way I was implicitly using in my previous posts, is to limit consideration to states that are physically realizable, which are also normalizable. So the postulates would read like this (note the bolded additions):
  • Physically realizable quantum states are unit vectors in some Hilbert space
  • Observables are Hermitian operators acting on said Hilbert space which have eigenstates that are physically realizable
  • Probability of results of physically realizable measurements are calculated from inner products between the system's current state and the observable's eigenstates
The "states" ##e^{ipx}## are not physically realizable states, and the momentum operator that has those "states" as its eigenstates is not a physically realizable operator (since it would correspond to measuring the momentum with infinite exactness, and physically realizable measurements only have finite accuracy). So the above postulates do not apply to them.

The other method of dealing with the technical issue is to use, instead of just Hilbert space, something called "rigged Hilbert space". This is basically an expansion of Hilbert space to include non-normalizable "states" like ##e^{ipx}## and operators that have those states as eigenstates, with a lot of technical precautions taken to ensure that physically realizable measurements still behave properly. If we take that route, then the postulates would read like this:
  • Quantum states are unit vectors in some rigged Hilbert space, which must be normalizable if they correspond to physically realizable states
  • Observables are Hermitian operators acting on said rigged Hilbert space
  • Probability of results are calculated from inner products between the system's current state and the observable's eigenstates, and then applying corrections as necessary if the states involved are not normalizable
Either approach is valid, but you have to be very careful not to mix the two.
 
  • Love
  • Like
  • Informative
Likes dextercioby, vanhees71, Lagrange fanboy and 2 others
  • #14
Lagrange fanboy said:
But the norm of the wavefunction won't be 1 then, shouldn't this cause a problem down the line when calculating probabilities?
That's among the most difficult subtleties in the beginning of learning quantum theory. If you have self-adjoint operators with a continuous spectrum the corresponding "eigenfunctions" almost always are not normalizable to 1 and thus do not belong to the Hilbert space. The math was also a problem in the beginning of quantum theory, and the mathematicians were inspired by Dirac's invention of his distribution to develop modern functional analysis.

The pragmatic physicist's answer is that in this case the "eigenfunctions" are "normalized to a ##\delta## distribution", i.e.,
$$\langle p|p' \rangle=\int_{\mathbb{R}} \mathrm{d} x u_p^*(x) u_p(x).$$
Now you have
$$\hat{p} u_p(x)=-\mathrm{i} \hbar u_p'(x)=p u_p(x) \; \Rightarrow \; u_p(x)=A(p) \exp(\mathrm{i} p x/\hbar).$$
From the theory of Fourier integrals indeed you get
$$\langle p|p' \rangle=\int_{\mathbb{R}} \mathrm{d} x A^*(p) A(p') \exp[\mathrm{i} x (p'-p)/\hbar] = 2 \pi |A(p)|^2 \delta[(p-p')/\hbar] = 2 \pi \hbar |A(p)|^2 \delta(p-p').$$
To "normalize the momentum eigenfunction to the ##\delta## distribution" thus implies that you can set
$$A(p)=\frac{1}{\sqrt{2 \pi \hbar}.$$
The normalization factor is only determined up to an arbitrary phase factor, which is unimportant, i.e., you can take the above most simple choice for the normalization factor without loosing anything.

If you want to learn about the "rigged Hilbert space" mentioned in the previous posting, a good starting point is the textbook

L. E. Ballentine, Quantum Mechanics, World Scientific,
Singapore, New Jersey, London, Hong Kong (1998).
 
  • Like
Likes PeroK and topsquark

1. What is the purpose of normalizing a wave function?

The purpose of normalizing a wave function is to ensure that the total probability of finding a particle in any possible location is equal to 1. This is necessary for the wave function to accurately describe the behavior of a particle.

2. How do you calculate the normalization factor of a wave function?

The normalization factor can be calculated by taking the square root of the integral of the absolute square of the wave function over all possible values of the independent variable. This ensures that the total probability is equal to 1.

3. Can a wave function be normalized to a value other than 1?

No, the normalization factor of a wave function must always be equal to 1. This is a fundamental property of wave functions and is necessary for them to accurately describe the behavior of particles.

4. Why is it important to normalize a wave function?

Normalizing a wave function is important because it ensures that the total probability of finding a particle in any location is equal to 1. This allows the wave function to accurately describe the behavior of particles and is necessary for many calculations in quantum mechanics.

5. Is it possible for a wave function to be non-normalizable?

Yes, it is possible for a wave function to be non-normalizable. This means that the integral of the absolute square of the wave function over all possible values of the independent variable does not converge. In such cases, the wave function cannot accurately describe the behavior of particles and is not considered a valid solution in quantum mechanics.

Similar threads

Replies
24
Views
532
Replies
1
Views
1K
  • Quantum Physics
Replies
3
Views
2K
  • Quantum Physics
Replies
2
Views
1K
Replies
1
Views
615
  • Quantum Physics
Replies
19
Views
1K
Replies
2
Views
536
Replies
5
Views
1K
  • Advanced Physics Homework Help
Replies
1
Views
1K
Replies
3
Views
623
Back
Top